LSAT and Law School Admissions Forum

Get expert LSAT preparation and law school admissions advice from PowerScore Test Preparation.

 jessamynlockard
  • Posts: 42
  • Joined: Jan 15, 2018
|
#45967
Hi,
I had a little difficulty getting to the answer of C.

Because of my psychology background, answer choice A is appealing, just because it would be a really important revelation when considering research on treatment efficacy.

B is a bit appealing because the stimulus' conclusion suggests that long-term treatment is unwarranted because it isn't better and is more expensive.

Would you mind explaining why A & B are wrong and C is right?
 Alex Bodaken
PowerScore Staff
  • PowerScore Staff
  • Posts: 136
  • Joined: Feb 21, 2018
|
#46009
jessamylockard,

Thanks for the question! This is a weaken question, meaning that the correct answer will serve to weaken the conclusion. So what is the conclusion? The author states the premises and then closes by saying the conclusion "Thus, for most people the generally more expensive long-term training is unwarranted." In other words, the author is concluding that there usually isn't a reason for people to get long-term treatment for anxiety. We should be able to have a really strong prephrase here: if the conclusion is that long-term treatment is usually unwarranted, we should expect the correct answer choice to provide us with a reason or explanation that suggests that long-term treatment is warranted/useful/desirable. With that idea in our heads, let's move to the answer choices.

Answer choice (A) reads: "A decrease in symptoms of anxiety often occurs even with no treatment or intervention by a mental health professional." If this were true, it suggests that perhaps no treatment is warranted or necessary - but that doesn't weaken our conclusion that long-term treatment is ineffective (if anything, it strengthens it, as long-term treatment is still a form of treatment). Therefore, it is not our credited answer. Be really careful - it sounds like your psychology background may have tripped you up a bit here :-D Remember that the LSAT does not require any prior, specialized knowledge of us - so even if we know a statement like answer choice (A) to be true, that is irrelevant - all that matters is if, if it were true, it would weaken the conclusion. It does not, and so it is incorrect.

Answer choice (B) reads: "Short-term relaxation training conducted by a more experienced practitioner can be more expensive than long-term training conducted by a less experienced practitioner." This answer choice suggests that in some cases, short-term relaxation training may be more expensive than long-term training, but does not get to the core issue of training effectiveness. Because this answer choice doesn't suggest that long-term training could be effective, it is not the credited answer choice.

Answer choice (C) reads: "Recipients of long-term training are much less likely than recipients of short-term training to have recurrences of problematic levels of anxiety." Here we go: this is a clear reason that long-term training might be preferable to short-term training (it leads to fewer reoccurrences of anxiety). Because this answer choice directly provides us with a reason that long-term treatment might be warranted, it attacks and weakens the conclusion, and is our credited answer.

Hope that helps!
Alex

Get the most out of your LSAT Prep Plus subscription.

Analyze and track your performance with our Testing and Analytics Package.